LSAT and Law School Admissions Forum

Get expert LSAT preparation and law school admissions advice from PowerScore Test Preparation.

User avatar
 Dave Killoran
PowerScore Staff
  • PowerScore Staff
  • Posts: 5852
  • Joined: Mar 25, 2011
|
#72706
This game is also discussed in our Podcast, at the 24:24 mark: LSAT Podcast Episode 37: The November 2019 LSAT Logic Games Section


Complete Question Explanation

(The complete setup for this game can be found here: https://forum.powerscore.com/lsat/viewtopic.php?t=31745)

The correct answer choice is (E).

There can be six ingredients only under the first template in this game:


Template #1: J is selected for the recipe

When J is included, it must be first or last, and Y cannot be an ingredient. Consequently, P must be included as an ingredient:

  • J P G K O T .....       Y
    ..... In ..... ..... Out


    ..... ..... ..... ..... ..... ..... ..... ..... :longline: K
    ..... ..... ..... ..... ..... J/ ..... O :longline: G ..... ..... /J
    ..... ..... ..... ..... ..... ..... ..... ..... :longline: T

    ..... ..... ..... ..... ..... ..... ..... ..... P?
Using the Grouping setup, you can see that Y cannot be selected, and thus answer choice (E) is correct.

Alternately, using the sequence, the following four ingredients could be added sixth (last): J, P, K, or T. That eliminates answer choices (A), (B), (C), and (D).


Answer choice (E): This is the correct answer. Y cannot appear in a recipe of six ingredients, and thus can never be sixth.

Get the most out of your LSAT Prep Plus subscription.

Analyze and track your performance with our Testing and Analytics Package.